Neuro MKSAP

Ace your homework & exams now with Quizwiz!

Plasma Exchange Therapy. This patient is exhibiting signs and symptoms consistent with idiopathic transverse myelitis (TM). Idiopathic TM is a monophasic inflammatory and demyelinating myelopathy affecting a portion of the spinal cord. Affected patients frequently experience a subacute onset of weakness, sensory changes, and bowel or bladder dysfunction, which is sometimes preceded by back pain or a thoracic banding sensation. Diagnostic criteria for idiopathic TM require the presence of clinical features of the syndrome, evidence of inflammation (either leukocytosis in the cerebrospinal fluid or contrast enhancement on MRI), and exclusion of other potential causes. First-line treatment for this disorder is administration of high-dose intravenous glucocorticoids. A 5-day course of high-dose methylprednisolone, however, had no beneficial effect on this patient's symptoms. The most appropriate next step is plasma exchange therapy, which has been shown to improve outcomes in patients with idiopathic transverse myelitis that is refractory to glucocorticoids.

A 19-year-old man is evaluated in the hospital after admission for subacute onset of bilateral lower extremity paraplegia, urinary incontinence, and sensory deficits. His initial treatment was a 5-day course of high-dose intravenous methylprednisolone. Four days after completion of the infusions, no clinical improvement has occurred. The patient had flu-like symptoms for several days before onset of neurologic symptoms but otherwise has been healthy. On physical examination, vital signs are normal. Muscle strength is 0/5 in both legs and 5/5 in both arms. Reflexes are absent in the lower extremities and normal in the upper extremities. Moderate sensory loss is noted below T3 bilaterally. Cerebrospinal fluid analysis: Erythrocyte count 2/µL (2 × 106/L) ,Leukocyte count 38/µL (38 × 106/L), with a predominance of lymphocytes ,Glucose Normal,Protein 62 mg/dL (620 mg/L). A T2-weighted MRI of the thoracic spine shows a hyperintense lesion in the thoracic cord at T2 with peripheral contrast enhancement. An MRI of the brain is normal.Which of the following is the most appropriate next step in treatment?

Focal seizures. The patient is experiencing focal seizures with altered awareness (formerly known as complex partial seizures or focal dyscognitive seizures). These seizures typically are infrequent, are associated with "warning" symptoms (aura, which may consist of an epigastric rising sensation or a feeling of déjà vu), last more than 30 seconds, have associated mouth or limb automatisms (semipurposeful repetitive movements), and are followed by confusion and/or exhaustion. Patients who experience this type of seizure often have no memory of the episode itself.

A 19-year-old woman is evaluated for a 6-month history of recurrent episodes of confusion that occur approximately once monthly. Her boyfriend says she has periods of wide-eyed staring, chewing motions, and repetitive grabbing of her clothes with the right hand. The patient sometimes experiences a strange but familiar feeling before the episodes but does not remember the episodes themselves, which last approximately 45 to 60 seconds and are followed by exhaustion and sleepiness for 20 minutes.All physical examination findings are normal. Which of the following is the most likely diagnosis?

Neuropsychological testing. She sustained a mild closed head injury with typical symptoms of a concussion. Loss of consciousness established the presence of mild traumatic brain injury (TBI). Headache, nausea, dizziness, and cognitive dysfunction are all typical symptoms of postconcussion syndrome. Guidelines recommend initial screening with a symptom checklist and a neurologic examination, including assessments of cognition and balance. Guidelines also recommend head CT in those with suspected hemorrhage. Neuropsychological testing provides an additional objective and more sensitive measure of cognitive function and should be part of a comprehensive TBI management strategy for patients with persistent symptoms.

A 20-year-old woman comes to the office for follow-up evaluation 2 weeks after being involved in a head-to-head collision in a soccer game. She lost consciousness for several seconds on the field and was removed from play. On the sidelines, she reported a severe occipital headache, dizziness, "brain fog," and nausea. Subsequent neurologic examination and head CT findings were unremarkable. She was started on a 1-week course of ibuprofen and prochlorperazine, which she took regularly. Besides mild cognitive slowing and short-term memory issues, she has been completely asymptomatic without medication for 1 week. On physical examination, blood pressure is 96/66 mm Hg and pulse rate is 62/min. All other physical examination findings, including those from a neurologic examination, are normal. Which of the following is the most appropriate next step in management?

Valproic Acid. The patient should now receive intravenous valproic acid. He is in generalized convulsive status epilepticus (CSE). A medical emergency that can lead to significant morbidity and mortality, CSE is defined as persistent tonic-clonic activity with impaired mental status that lasts longer than 5 minutes. Initial management of CSE requires rapidly assessing airways, breathing, and circulation; checking the blood glucose level; and administering thiamine with glucose, if needed. These steps should be performed simultaneously with initiation of drug treatment. According to a guideline from the American Epilepsy Society, intramuscular midazolam, intravenous (IV) lorazepam, and IV diazepam have been judged to be equivalent and are the first-line agents for the initial treatment of CSE. Phenytoin, a longer-acting antiseizure drug, should then be administered; if available, fosphenytoin, a prodrug of phenytoin, is preferable to phenytoin for initial treatment of CSE because it can be administered faster and does not carry the risk of thrombophlebitis or skin necrosis (purple glove syndrome) that is associated with phenytoin extravasation. Valproic acid is an alternative to phenytoin or fosphenytoin, particularly in patients with an allergy to phenytoin or primary generalized epilepsy. Therefore, intravenous valproic acid should be administered to this patient as a second-line therapy.

A 21-year-old man is evaluated in the emergency department for persistent convulsive status epilepticus that began 30 minutes before his arrival. An airway has been secured, and he has received intravenous glucose, thiamine, and two doses of intravenous lorazepam. After receiving the second dose of lorazepam, he continues shaking for another 5 minutes. Medications are levetiracetam and acetaminophen; the patient is allergic to phenytoin. On physical examination, temperature is normal, blood pressure is 155/89 mm Hg, pulse rate is 108/min, respiration rate is 16/min, and oxygen saturation with the patient breathing 6 L of oxygen via a nasal cannula is 97%. The pupils are reactive but he remains comatose. Results of a urine drug screen are negative. Which of the following is the most appropriate next step in management?

Acetaminophen. The patient has symptoms and signs of a severe head injury. Severe headache combined with nausea and focal neurologic symptoms, such as tinnitus and hearing loss, raises concerns of a major cranial injury. Bilateral periorbital bruising ("raccoon eyes"), mastoid bruising ("Battle sign"), and hemotympanum suggest the presence of a basilar skull fracture. Cerebrospinal fluid (CSF) rhinorrhea, epistaxis, and cranial nerve palsies also can be noted with these fractures, and fractures of the temporal bone occur in 75% of affected patients. Maintenance of an arterial PO2 level of greater than 60 mm Hg (8.0 kPa) and a systolic blood pressure of greater than 90 mm Hg have been shown to improve outcomes in these patients. Fever worsens outcomes after stroke and possibly after severe head injury, most likely by promoting secondary brain injury, and acetaminophen is the most appropriate treatment at this time.

A 22-year-old man is evaluated in the hospital after sustaining a traumatic brain injury in a motorcycle accident 4 hours earlier. The patient lost consciousness briefly at the scene but was wearing a helmet. He is now awake and reports a severe headache, nausea, vertigo, left-ear tinnitus and deafness, and rhinorrhea when he sits upright. On physical examination, temperature is 38.1 °C (100.6 °F), blood pressure is 110/60 mm Hg, pulse rate is 104/min, and respiration rate is 20/min. Oxygen saturation is 95% with the patient breathing ambient air. Right hemotympanum and deafness are noted. Bruising is shown. Results of laboratory studies show a normal complete blood count, comprehensive metabolic profile, and urinalysis; a urine drug screen is positive for opiates. A noncontrast head CT scan shows a small right temporal bone fracture but no intracranial hemorrhage. Chest radiographs and a cervical spine CT scan are normal. Which of the following is the most appropriate initial treatment?

Metoprolol. Her headaches meet the criteria for episodic migraine without aura. Pharmacologic prevention should be considered for migraine occurring at a frequency of at least 5 days per month. According to guidelines, metoprolol, propranolol, timolol, divalproex sodium, and topiramate all have Level A evidence for prevention of episodic migraine, with atenolol, amitriptyline, and venlafaxine having level B evidence. Approximately a third of patients with episodic migraine meet criteria for pharmacologic prevention, yet only 3% to 13% receive this treatment. The goal of prevention is an at least 50% reduction in headache frequency; reductions in migraine intensity, disability, and cost are other established outcomes. Drug tapering or elimination should be considered after a period of 6 to 12 months of adequate control.

A 24-year-old woman has a 4-year history of monthly headaches. The pain is bitemporal, throbbing, and worsened by bending or ascending stairs. Approximately half of the headaches have been severe with associated nausea. She has no precursor symptoms, no photophobia or phonophobia, and no visual or neurologic symptoms. Over the past year, the headaches have not changed in character but have gradually become more frequent, increasing from 4 to 10 days per month. Episodes last as long as 8 hours and typically respond to ibuprofen within 1 hour. She takes no other medication. All physical examination findings, including vital signs and those from a neurologic examination, are unremarkable. Which of the following is the most appropriate treatment?

Epidural hematoma. The patient has an epidural hematoma. This head injury most commonly results from a fracture of the temporal bone and subsequent laceration of the ipsilateral middle meningeal artery. Only 20% to 30% of these hematomas occur in other locations, and most follow trauma; spontaneous hemorrhages are rare. An epidural hematoma develops between the inner table of the skull and the dura mater. Lateral extension is prevented by dual attachments at the skull sutures; expansion occurs inwardly towards the brain parenchyma. Appearance on a head CT scan is biconvex or "lenticular." After a lucid interval, neurologic deterioration is rapid. Peak expansion of the hematoma occurs 6 to 8 hours after initial formation. Severe headache, nausea with vomiting, and eventual impairment in consciousness progress quickly. Ipsilateral compression of the oculomotor nerve (cranial nerve III) indicates potential uncal herniation. Subsequent ipsilateral midbrain compression results in contralateral upper motor neuron findings. Emergent surgical evacuation is required for those with anisocoria, a Glasgow Coma Scale (GCS) score of less than 9, or a hematoma volume greater than 30 mL.

A 25-year-old woman is evaluated in the emergency department (ED) 90 minutes after being struck in the right temple by a foul ball at a baseball game. Although knocked from her seat and initially dazed by the blow, the patient continued to watch the game for another 45 minutes until she developed a worsening headache with nausea and vomiting. On arrival in the ED, the patient was stuporous and became unresponsive during subsequent CT of the head. On physical examination, blood pressure is 140/90 mm Hg, pulse rate is 100/min, and respiration rate is 14/min. The right pupil is dilated. Right oculomotor nerve (cranial nerve III) palsy and left hemiparesis are noted. A plantar extensor response is present on the left. A CT scan of the head is shown. Which of the following is the most likely diagnosis?

Convulsive syncope. Syncope is nontraumatic, complete transient loss of consciousness and loss of postural tone. Onset is abrupt and recovery is spontaneous, rapid, and complete. Neurally mediated syncope, the most common type of syncope, generally occurs with standing and is associated with a prodrome of nausea, lightheadedness, and warmth. It may follow cough, urination, defecation, pain, or laughing. This patient's tunnel vision, palpitations, short duration of loss of consciousness (<1 minute), and immediate and complete neurologic recovery is typical of syncope. Movements and shaking (in this instance, nonepileptic myoclonus) are common with syncope. In fact, syncope without any movements is the exception rather than the rule.

A 27-year-old woman is evaluated in the emergency department after a sudden, first-time episode of loss of consciousness while standing in line to board a tour bus. She had warning symptoms of tunnel vision and palpitations, after which she lost consciousness and fell. According to her father, who witnessed the episode, she was limp and unconscious for approximately 20 to 30 seconds, during which time she displayed intermittent twitching of all four limbs, with the limbs shaking independently at separate times. After the patient regained consciousness, she was confused about why she was on the ground but answered questions appropriately and was oriented to self and place. All physical examination findings are normal. Which of the following is the most likely diagnosis?

Primary stabbing headache. (also known as an "ice-pick" headache). The condition is characterized by transient localized stabs of head pain that occur spontaneously in the absence of organic disease and typically last seconds, with some lingering for 1 to 2 minutes; a less-localized dull ache or soreness lasting minutes may follow. The frequency of primary stabbing headache averages less than one per day in most patients, but headache attacks also can occur in series. Periods of activity and remission are common. Pain can occur anywhere on the head, including the eye, but the face is often spared. Primary stabbing headache is more common among those with a history of migraine. No cranial autonomic symptoms are reported. Indomethacin may be helpful during cycles of repeated occurrences but is rarely necessary.

A 30-year-old man is evaluated for a 6-week history of repeated episodes of sharp pain isolated to the right parietal region lasting less than 10 seconds. The pain is intense, begins and ends spontaneously without a clear trigger, and can recur multiple times each day, sometimes in series. The patient has a history of migraine with aura from age 6 through 12 years but has had no headaches since that time. He has had no associated autonomic features and no symptoms of migraine. He has a family history of migraine with aura. The patient takes no medication. All physical examination findings, including vital signs and those from a neurologic examination, are unremarkable. An MRI of the brain is normal. Which of the following is the most likely diagnosis?

Autoimmune limbic encephalitis. New-onset status epilepticus in a previously healthy person suggests the presence of a neurologic autoimmune disorder. This patient has a rapidly progressive neurologic syndrome that is most consistent with autoimmune encephalitis, with antibodies directed against the voltage-gated potassium channel receptor complex (the most common target being the LGI1 antibody). This condition is marked by the presence of hyponatremia, myoclonus, and limbic encephalitis (amnesia, temporal lobe seizures, and confusion). Although many neurologic autoimmune disorders are paraneoplastic, primary autoimmune disorders with no accompanying cancer may occur.

A 33-year-old man is evaluated in the emergency department for a 10-week history of worsening confusion, memory loss, and difficulty speaking. He has become progressively more disorganized at work and can no longer complete routine tasks. His medical history is otherwise unremarkable, and he takes no medication. On physical examination, vital signs are normal. The patient exhibits decreased attention, is able to follow only simple commands, and is oriented only to person. Speech is dysarthric. Intermittent myoclonic jerking of varying limbs also is noted. Results of laboratory studies show a serum sodium level of 128 mEq/L (128 mmol/L) but are otherwise unremarkable, including a normal complete blood count, comprehensive metabolic panel, serum thyroid-stimulating hormone level, and free thyroxine (T4) level. An urgent electroencephalogram shows evidence of nonconvulsive status epilepticus with focal seizures arising independently from both temporal lobes. A contrast-enhanced brain MRI is normal. Which of the following is the most likely diagnosis?

Lamotrigine. is the most appropriate antiepileptic drug (AED) to treat this patient's focal seizures. Carbamazepine, phenytoin, and phenobarbital are all inducers of the cytochrome p450 system; these drugs increase breakdown of vitamin D, which results in increased parathyroid hormone levels, and thus cause bone loss and osteoporosis. Valproic acid also is associated with bone loss, although the mechanism of this effect is unclear. Lamotrigine and other AEDs that do not induce the P450 system, such as levetiracetam, have no recognized effect on bone turnover; therefore, of the choices listed, lamotrigine is the most appropriate AED to use in a patient with established osteoporosis.

A 35-year-old man is evaluated for two breakthrough focal seizures in the past week. He started having focal and generalized tonic-clonic seizures 20 years ago. The seizures previously had been well controlled by phenytoin, which he has taken since age 16 years, but he missed three doses of medication this week. He has had no recent illnesses or fevers. On physical examination, all findings, including vital signs, are normal. Results of bone densitometry show osteoporosis (T-score of −2.8). In addition to starting the patient on alendronate, which of the following drugs should be substituted for the phenytoin?

Lamotrigine. No single antiepileptic drug (AED) is recommended for the initial treatment of epilepsy. Approximately 50% of patients with this disorder will respond to the first AED administered. Choosing an AED for an individual patient depends on several factors, including his or her epilepsy syndrome, age, sex, and comorbid medical conditions and the drug's adverse-effect profile and cost. Lamotrigine is commonly prescribed in women with childbearing potential and is also a good option for older patients or those who have depression or other mood disorders. This patient has generalized epilepsy that is not controlled with valproic acid. Lamotrigine is the most appropriate medication to use instead, not only because of its effectiveness as treatment for generalized tonic-clonic seizures, but also because of its mood-stabilizing effects. Lamotrigine is FDA approved for maintenance treatment of bipolar I disorder, and clinical trials have demonstrated its efficacy in the treatment of depersonalization/derealization disorder.

A 38-year-old man comes to the office for a routine follow-up evaluation of generalized tonic-clonic seizures, which he has experienced for the past year. He has had no improvement in seizures despite treatment with maximal doses of valproic acid. The patient also has depression with psychotic features. He attempted suicide 3 years ago. He has no additional medical problems. His only other medication is citalopram. In addition to discontinuing the valproic acid, which of the following is the most appropriate treatment?

Glatiramer acetate. He has newly diagnosed multiple sclerosis (MS) and should begin a disease-modifying therapy. Definitive treatment of MS relies less on addressing relapses as they occur than on preventing relapse occurrence (and the associated accrual of disability) in the first place. This prevention is achieved with MS disease-modifying therapies, a series of immunomodulatory or immunosuppressive medications that have been shown to reduce the risk of relapse, disability progression, and new lesion formation on MRI. Several disease-modifying therapies have been approved by the FDA for use in relapsing-remitting MS, each differing in their route of administration, mechanism of action, and potential adverse effects. Generally, most physicians recommend self-injection medications (one of the interferon beta preparations or glatiramer acetate) as first-line agents, given their favorable risk profiles. Glatiramer acetate, a copolymer of four amino acids, is administered daily or several times weekly by subcutaneous injection. Glatiramer acetate and high-dose interferon beta formulations exhibit similar reductions in relapse rates compared with placebo and are equivalent in head-to-head studies. In choosing which therapy is most appropriate to recommend, the clinician should consider all comorbid conditions. Given the patient's history of liver disease, the most appropriate treatment is glatiramer acetate. This medication has no known adverse effects on liver function.

A 38-year-old man comes to the office to discuss treatment of his recently diagnosed multiple sclerosis. The patient also has diabetes mellitus and nonalcoholic steatohepatitis. Medications are metformin and atorvastatin. On physical examination, vital signs are normal; BMI is 30. All remaining physical examination findings are unremarkable. A T2-weighted MRI of the brain obtained just before diagnosis shows three hyperintense lesions in the periventricular white matter, a hyperintense lesion in the pons, and three juxtacortical hyperintense lesions. The pontine lesion was enhanced with administration of intravenous contrast. Which of the following is the most appropriate treatment?

Sumatriptan. He has a history of episodic migraine with typical aura that is no longer responsive to appropriate NSAIDs. Guidelines recommend the use of triptans in patients with moderate to severe migraine for whom NSAID therapies are not effective. Triptans (5-hydroxytryptamine receptor 1B [5-HT1B] and 5-hydroxytryptamine receptor 1D [5-HT1D] agonists) are migraine-specific agents with a direct impact on the trigeminovascular activation associated with migraine attacks. Activation of the 5-HT1B receptor reverses vasodilation. Binding to the 5-HT1D receptors on trigeminal nerve terminals blocks release of vasoactive and inflammatory mediators, such as calcitonin gene-related peptide. This second action is likely more important in the interruption of nociceptive transmission. Unless medication overuse headache is a concern, patients should be advised to treat at the first sign of pain. Because migraine intensity is highly variable, occasional attacks that are less responsive to medication are to be expected. Contraindications include coronary, cerebral, or peripheral vascular disease; uncontrolled hypertension; and migraine with brainstem or hemiplegic auras.

A 38-year-old man is evaluated for a 6-month history of increasingly frequent episodes of migraine with aura. The patient has had migraine with aura since age 23 years. Auras involve 15 to 20 minutes of twinkling lights and visual blurring in either hemifield. On occasion, he has noted ipsilateral numbness and paresthesia of the face, tongue, and hand lasting another 15 to 20 minutes after the visual blurring has resolved. Migraine episode frequency has increased from once to twice monthly. Acetaminophen, ibuprofen, and naproxen were each discontinued after becoming ineffective, with headaches now lasting 24 hours. On physical examination, all findings, including vital signs and results of a neurologic examination, are unremarkable. Which of the following is the most appropriate next step in treatment?

MRI of the brain. She has signs and symptoms consistent with an episode of inflammatory transverse myelitis. Although idiopathic transverse myelitis can occur, the most common cause of a transverse myelitis event is multiple sclerosis (MS). MRI of the brain should be routinely performed in patients with new-onset transverse myelitis to evaluate for lesions consistent with MS, such as those located in periventricular, juxtacortical (or cortical), and infratentorial white matter. If these lesions are present, MS should be diagnosed if they fulfill the diagnostic criteria for dissemination in space and in time.

A 39-year-old woman is evaluated in the emergency department for a 2-day history of bilateral lower extremity weakness and numbness. The patient has no other symptoms and no relevant medical history. She takes no medication. On physical examination, temperature is 36.7 °C (98.1 °F); all other vital signs are normal. Muscle strength testing shows 4/5 weakness in hip flexion bilaterally and in foot dorsiflexion. Temperature sensation is partially reduced below spinal cord level T5. All other physical examination findings are normal. A T2-weighted MRI of the thoracic spine shows a hyperintense lesion at the T4 level. Which of the following is the most appropriate next test to evaluate for multiple sclerosis?

High Dose Oral Prednisolone. This patient should receive high-dose oral prednisone, 1250 mg/d for 5 days. She is experiencing an acute exacerbation, or relapse, of multiple sclerosis (MS). Administration of high-dose glucocorticoids is the standard treatment for MS exacerbations. This treatment approach derives from the landmark Optic Neuritis Treatment Trial, in which high-dose intravenous methylprednisolone (1 g/d for 5 days) was compared to prednisone (1 mg/kg/d for 2 weeks) and placebo. Intravenous methylprednisolone was associated with a more rapid recovery of visual function. Subsequent studies have shown that use of a bioequivalent oral high-dose regimen (such as oral prednisone, 1250 mg, or oral methylprednisolone, 1 g) is as efficacious as intravenous methylprednisolone and has no significant differences in adverse effects. Because of their ease of administration and reduced costs, high-dose oral regimens are beginning to replace intravenous methylprednisolone, although either approach is valid.

A 39-year-old woman is evaluated in the emergency department for a 3-day history of worsening imbalance, falling, and vertigo. Multiple sclerosis was diagnosed 5 years ago and has been treated with interferon beta-1a since that time. She also takes gabapentin for neuropathic pain. On physical examination, temperature is 37.3 °C (99.1 °F); all other vital signs are normal. Internuclear ophthalmoplegia is noted on the left. The patient exhibits imbalance in her primary gait and is unable to perform tandem gait or to maintain balance during Romberg testing when she opens her eyes. Which of the following is the most appropriate immediate treatment?

Dystonic tremor. which occurs both at rest and with action and is characterized by associated dystonic posturing and the presence of a null point at which change in the position of the affected limb resolves the tremor. The null point is the position at which the trajectories of the forces caused by dystonic coactivation of agonist and antagonist muscles neutralize each other, which leads to resolution of the tremor. In addition, the action component of tremor has task specificity in that it is worse with use of scissors but spares the handwriting.

A 41-year-old woman is evaluated for a 2-year history of tremor in the dominant right hand. She says that the tremor has begun interfering with her work as a hairdresser, especially when she uses scissors. She also reports tightness in the forearm. The patient is able to eat, write, and type without difficulty and has had no trauma, imbalance, slowness of movement, or change in gait speed. Alcohol has no effect on the tremor. There is no family history of tremor. On physical examination, vital signs are normal. A right upper extremity tremor is noted, as are rhythmic flexion of the wrist, involuntary flexion of the fingers, and pronation of the forearm. The tremor is present both at rest and during action and resolves by changing the position of an outstretched arm. No dysmetria, dysdiadochokinesia, bradykinesia, rigidity, shuffling gait, or reduced arm swing is noted. Her handwriting is neither tremulous nor micrographic. An MRI of the brain is unremarkable. Which of the following is the most likely diagnosis?

Rapid eye movement (REM) sleep behavior disorder. commonly occurs in dementia with Lewy bodies (DLB) and can help distinguish DLB from Alzheimer disease and other cognitive disorders. This patient has cognitive and other symptoms suggestive of mild DLB. All the synucleinopathies (Parkinson disease, Parkinson disease dementia, and DLB) have a much higher rate of REM sleep behavioral sleep disorder than other neurologic and neurodegenerative diseases. REM sleep behavior disorder is characterized by the acting out of dreams secondary to loss of normal muscle paralysis during the dream phase of sleep. Symptoms may range from hand gestures to violent thrashing, punching, and kicking that may result in harm to self or bed partner. Patients also may have severe sensitivity to neuroleptic medications, which is more common in DLB than in other dementing illnesses.

A 59-year-old man is evaluated for recent cognitive problems resulting in worsening performance in his work as an electrician. He reports having increasing difficulty "running the numbers" when determining the materials needed for a job and increasing problems understanding the layout of plans when working on a new construction. He also notes occasional hand tremors when he works. According to his wife, his gait has slowed somewhat and he seems to have less of a response to things occurring around him in the past few weeks, stating that he "doesn't show any emotion in his face anymore." The patient also has a 5-year history of erectile dysfunction, a 4-year history of constipation, and episodes of lightheadedness when standing for long periods at work. On physical examination, vital signs are normal. A general paucity of movements is noted, as are slowness of movement in the upper and lower extremities and a mildly stooped posture when walking. He scores 20/30 (normal, ≥26) on the Montreal Cognitive Assessment, having significant difficulty with the cube drawing, clock drawing, and sustained attention portions of the test. Which of the following additional clinical findings would be most helpful in establishing the diagnosis in this patient?

Donepezil. Given her history of multiple strokes, pseudobulbar affect (a neurologic disorder characterized by involuntary outbursts of laughing and/or crying that are out of proportion to the emotions being experienced), prominent gait problems, and asymmetric neurologic findings, she most likely has vascular cognitive impairment. Treatment of vascular cognitive impairment should focus on identifying and treating cerebrovascular risk factors, such as smoking, diabetes mellitus, hyperlipidemia, hypertension, ischemic heart disease, atrial fibrillation, and hypercoagulable states. Although not yet FDA approved as treatment of vascular cognitive impairment, acetylcholinesterase inhibitors have shown modest benefit in clinical trials and are generally recommended for this type of dementia.

An 85-year-old woman is evaluated for declining function. According to her son, her memory has been failing, her gait has slowed, she has fallen four times, and she often cries without apparent reason. He further reports that he does most of her cooking and has assumed responsibility for her finances. The patient had a right basal ganglia infarct 5 years ago and left cerebellar infarct 2 years ago. She also has atrial fibrillation and hypertension. Medications are rivaroxaban, hydrochlorothiazide, and metoprolol. On physical examination, blood pressure is 155/80 mm Hg in both arms, and pulse rate is irregularly irregular at 90/min; other vital signs are normal. Cardiac examination shows findings consistent with atrial fibrillation. All other physical examination findings are normal. The patient says she has not been depressed but becomes tearful when discussing her medications. Depression screening has normal findings. She scores 20/30 (normal, ≥26) on the Montreal Cognitive Assessment. Bradykinesia with normal strength is noted in the legs bilaterally, as are increased deep tendon reflexes and an extensor plantar response on the left. Gait examination reveals a shuffling pattern. Results of laboratory studies show normal thyroid-stimulating hormone and vitamin B12 levels. An MRI shows diffuse white matter hyperintensities bilaterally and lacunar infarcts in the bilateral basal ganglia and left cerebellum. Which of the following is most appropriate for this patient's cognitive impairment?

Familial amyloidosis. His progressive sensory, motor, and autonomic axonal polyneuropathy; bilateral carpal tunnel syndrome; and family history of neuropathy and cardiac disease suggest familial amyloidosis. This disorder is diagnosed by detection of a transthyretin gene (TTR) mutation. Affected patients often have multiorgan involvement (diseases of the eye [glaucoma] and the neurologic, gastrointestinal, cardiac, and urinary systems), but there is wide phenotypic variation leading to delayed diagnosis. Therefore, a high level of clinical suspicion of amyloidosis is necessary in the presence of relentless progression of neuropathy, autonomic dysfunction, and positive family history of typical symptoms.

A 47-year-old man is evaluated for recent weakness and loss of balance during ambulation, multiple episodes of orthostatic syncope, and a 2-year history of burning paresthesia in the lower extremities. He also reports dry mouth and dry eyes. The patient has glaucoma, erectile dysfunction, constipation, and carpal tunnel syndrome. His father had sensory neuropathy, and his father and paternal uncle died of restrictive cardiomyopathy. Medications are gabapentin, sildenafil, and docusate sodium. On physical examination, blood pressure is 130/70 mm Hg sitting and 110/64 standing, and pulse rate is 110/min. Weakness in the distribution of the bilateral median nerves and in the proximal lower extremities is noted. Loss of sensation to pinprick is present in both hands and below the knees. Deep tendon reflexes are reduced in the lower extremities. Electromyography, including nerve conduction studies and needle electrode examination, reveals diffuse axonal sensorimotor polyneuropathy. Which of the following is the most likely diagnosis?

No further treatment is necessary. This patient requires no further treatment of his blood pressure at this time. He has had an acute ischemic stroke and was treated appropriately with recombinant tissue plasminogen activator (alteplase). His posttreatment blood pressure is less than 180/105 mm Hg, which is less than the guideline threshold for initiating antihypertensive therapy. Given the presence of internal carotid artery stenosis, expansion of the infarct is possible if the blood pressure is significantly lowered. Blood pressure lowering may be appropriate in patients with symptomatic intracerebral hemorrhage after thrombolysis, but this diagnosis is unlikely in this patient whose physical examination findings after thrombolysis are stable.

A 47-year-old man is evaluated in the emergency department 2 hours after sudden onset of right-sided face, arm, and leg weakness and numbness. He has a history of hypertension. Medications are amlodipine and hydrochlorothiazide. On physical examination, blood pressure is 166/72 mm Hg, pulse rate is 82/min, and respiration rate is 12/min. Right facial weakness, dysarthria, and loss of pinprick sensation in the right arm and leg are noted. The patient can lift the right arm and leg off the bed. The National Institutes of Health Stroke Scale score is 8 (moderate stroke). A CT scan of the head without contrast shows no hemorrhage or early signs of infarct. The patient receives intravenous recombinant tissue plasminogen activator (alteplase) 3 hours after symptom onset. One hour after treatment, blood pressure is 170/86 mm Hg; other physical examination findings are unchanged. A CT angiogram shows 80% stenosis of the left internal carotid artery and no intracranial arterial occlusion. Administration of which of the following is the most appropriate next step in treatment?

Oral Prednisolone. should be given to this patient with Bell palsy within 72 hours of symptom onset to expedite the speed and rate of full recovery. Clinical history and examination findings in this patient are classic for Bell palsy, and thus no initial imaging or laboratory testing is indicated. Unilateral facial weakness involving both upper and lower parts of the face is characteristic of peripheral facial nerve (cranial nerve VII) involvement and distinguishes this condition from a central nervous system process (such as stroke), which typically spares the upper facial muscles because of bilateral innervation. Associated features of classic Bell palsy include alteration in taste (due to involvement of the chorda tympani) and hyperacusis (intolerance of loud noise due to involvement of stapedius muscle); other common features are ipsilateral sensory paresthesia and pain without objective sensory loss. Because this patient does not have subjective or objective hearing loss or any other involvement beyond a unilateral facial nerve, initial brain imaging is not indicated.

A 48-year-old woman is evaluated for an 8-hour history of facial paralysis. She reports having "droopiness" on the right side of the face when she awoke this morning. She further says that her lunch tasted "odd" and that sounds on her right seem louder than usual. On physical examination, vital signs are normal. A right facial droop associated with difficulty closing the eye and elevating the eyebrow is noted on the right side. Hearing is intact to whispering, but noises are more pronounced on the right side. The palate is symmetrically elevated, and the tongue protrudes at the midline. Taste is impaired on the right side of the tongue and intact on the left side. Muscle strength is intact on the left side of the face and in both extremities. Shoulder shrug and head turning are normal bilaterally. Facial sensation is intact bilaterally, as are all deep tendon reflexes. The neck is supple, no skin rash is visible, and coordination and gait are normal. Which of the following is the most appropriate initial step in management?

Mild cognitive impairment. The patient's symptoms of subjective cognitive problems, near-normal function, and objective cognitive impairment (score of 1/5 on the delayed recall section of the Montreal Cognitive Assessment) are most consistent with a diagnosis of mild cognitive impairment (MCI), amnestic type. MCI is a cognitive state between normal aging and dementia characterized by a decline in cognitive functioning that is greater than what is expected with normal aging but has not resulted in significant functional disability. The suggestive MRI evidence of minimal hippocampal atrophy is a biomarker conferring a risk of symptom progression. Given her objective performance on cognitive tests and the noted MRI changes, the patient has an annual risk of developing Alzheimer disease of 5% to 15%.

A 72-year-old woman is evaluated for a 12-month history of increasing forgetfulness. The patient is retired from her position as a professor of economics at a local university. She takes no medication and drinks no alcoholic beverages. Her husband says that his wife's occasional forgetfulness has caused no major problems and that she is able to function normally. On physical examination, vital signs are normal. Her Geriatric Depression Scale score is normal and Montreal Cognitive Assessment score is 25/30 (normal ≥26), with a score of 1/5 on the delayed recall section and 5/6 on the orientation section. All other physical and neurologic examination findings are normal. Results of laboratory studies show normal vitamin B12, thyroid-stimulating hormone, and 25-hydroxyvitamin D levels. Complete neuropsychological testing findings indicate normal global cognition, language function, and attention, but verbal learning and memory performance are 2 SDs below that of an age- and education-matched control population. An MRI of the brain shows a slight loss of hippocampal volume that is greater on the right than the left but is otherwise normal. Which of the following is the most likely diagnosis?

Myoclonus. This patient's clinical history and clinical examination findings are most consistent with myoclonus. Myoclonus consists of rapid, nonsuppressible, shock-like, jerky movements that can result from metabolic, endocrine, toxic, infectious, epileptic, autoimmune, and other causes. Posthypoxic myoclonus (Lance-Adams syndrome) occurs in patients with a history of hypoxic brain injury (suggested in this patient by his previous cardiopulmonary arrest) and is characterized by prominent action-induced myoclonus that impairs ambulation because of a combination of positive (rapid jerky movements) and negative (lapses in muscle tone) myoclonus. Other examples of negative myoclonus include hiccups and asterixis.

A 49-year-old man is evaluated for a 6-month history of difficulty walking and frequent falls. He first noticed these symptoms after a prolonged hospitalization for pneumonia that was complicated by sepsis and cardiopulmonary arrest requiring intubation and antibiotic therapy. The patient also has hypothyroidism treated with levothyroxine. On physical examination, vital signs are normal; BMI is 35. Muscle strength is normal in all limbs. At rest, few rapid jerky movements are noted in various limbs. On standing, rapid, nonrhythmic, shock-like movements are noted in the trunk and lower extremities, and there are brief lapses of muscle tone in the legs, which lead to loss of balance. These movements are not suppressible. Cognition, cranial nerves, reflexes, and sensation are normal. Plantar response is flexor bilaterally, and clonus is absent. An MRI of the brain is unremarkable. Which of the following most accurately describes this patient's abnormal movements?

Aspirin. She most likely has a cardioembolic stroke related to atrial fibrillation. She is beyond the treatment window for intravenous thrombolysis and is not a candidate for intra-arterial therapy because of the location of the lesion in the posterior circulation and already demonstrated infarction. Therefore, medical therapy is indicated for preventing recurrent stroke. Only aspirin and heparinoids have been studied for their effectiveness in the acute stroke setting, and only aspirin has been shown to reduce the risk of stroke at 2 weeks. Therefore, aspirin monotherapy is a reasonable first-line antiplatelet regimen for secondary stroke prevention in the acute setting. For stable patients with small strokes, long-term warfarin therapy is typically administered 24 hours after hospitalization to reduce the risk of hemorrhagic transformation. Hemorrhagic transformation is more likely to occur in larger infarcts (those involving more than one third of hemispheric volume) and in infarcts with a cardioembolic cause. Warfarin is often withheld for 2 weeks in the setting of large strokes, strokes that have undergone hemorrhagic transformation, or uncontrolled hypertension.

A 49-year-old woman is evaluated in the emergency department for a 12-hour history of new-onset right-sided visual loss. The patient has type 2 diabetes mellitus treated with metformin. On physical examination, blood pressure is 138/72 mm Hg, and pulse rate is 102/min and irregularly irregular. Cardiac auscultation reveals no carotid bruits or cardiac murmurs. A visual field deficit is present on the right side of both eyes. No weakness or sensory loss is noted. A CT scan of the head shows a hypodensity in the left occipital lobe, and a carotid duplex ultrasound shows less than 60% stenosis of both internal carotid arteries with normal vertebral artery flow. An electrocardiogram shows atrial fibrillation. Which of the following medications should be administered now?

Carotid duplex ultrasonography. He most likely has had a transient ischemic attack (TIA) referable to the left hemisphere and is at risk of ischemic stroke within the next 90 days, with the highest risk occurring within the first 2 days. Although his ABCD2 score (based on patient Age, Blood pressure, Clinical presentation, Duration of symptoms, and presence of Diabetes mellitus) of 3 indicates a predicted stroke risk of 1.3% and the need for hospitalization and rapid evaluation, these scores are neither sensitive nor specific enough to identify patients at highest risk of stroke. Several studies have identified extracranial symptomatic internal carotid artery stenosis as an indicator of stroke risk of greater than 70% after TIA; this risk can be modified with carotid revascularization. Several modalities are available for imaging the internal carotid artery and identifying extracranial symptomatic stenosis, including duplex ultrasonography, CT angiography, and magnetic resonance angiography (MRA). Duplex ultrasonography has the benefit of being inexpensive, readily available, low risk, and noninvasive and thus is an appropriate early study. If the carotid duplex ultrasound shows high-grade stenosis, confirmation of this finding by CT angiography or MRA is required before surgical intervention. If the carotid duplex ultrasound is unrevealing, then additional vessel imaging of the neck is unnecessary.

A 49-year-old-man is evaluated 1 day after having an episode of right arm weakness without pain that lasted 5 minutes. He is now asymptomatic. The patient has type 2 diabetes mellitus and dyslipidemia. Medications are aspirin, metformin, and atorvastatin. On physical examination, blood pressure is 126/68 mm Hg, pulse rate is 86/min and regular, and respiration rate is 12/min. No carotid bruits or cardiac murmurs are heard on cardiac auscultation. All other physical examination findings are normal. An electrocardiogram shows normal sinus rhythm with no ST-segment or T-wave changes. Which of the following is the most appropriate initial imaging test?

Polysomnography. He has minor symptoms of cognitive impairment, such as memory lapses and calculation difficulties at work. To exclude a slowly progressive dementia and identify any potentially reversible causes of his symptoms, the patient's examination should consist of a neurologic examination, including a cognitive screening evaluation, and evaluation for depression, sleep disorders, alcohol abuse, and family history of dementia. The patient does not use alcohol and has no family history of dementia. His score on the Montreal Cognitive Assessment is borderline abnormal, which would be unlikely if Alzheimer disease or another progressive dementia were the cause of his symptoms, and several depression scales provide no evidence of depression. Because depression and a primary neurodegenerative disorder are unlikely in this patient, additional diagnostic testing is necessary. His hypertension, obesity, and self-reported daytime sleepiness are consistent with a diagnosis of obstructive sleep apnea (OSA). Other neuropsychiatric symptoms of OSA include mood alterations, difficulty concentrating, and problems completing tasks at school or the workplace. Objective testing is required for the diagnosis of OSA.

A 51-year-old man is evaluated for morning headaches and increasing difficulty with focus and memory. He reports growing problems at work, including making occasional mistakes when updating spreadsheets, missing three recent appointments, and occasionally falling asleep during staff meetings. The patient has no other medical problems. He does not drink alcoholic beverages, smoke, or have a family history of dementia. On physical examination, blood pressure is 150/90 mm Hg but other vital signs are normal; BMI is 33. He scores 25/30 (normal, ≥26) on the Montreal Cognitive Assessment, scoring 2/5 in the recall section, 0/1 in word generation, and 0/1 in letter identification. He scores normally on several clinical depression scales. Other physical examination findings, including those from a neurologic examination, are unremarkable. Which of the following is the most appropriate next step in evaluating this patient?

Modafinil. Chronic fatigue is a common symptom in multiple sclerosis (MS). The fatigue associated with MS can have various causes, such as depression, insomnia, or other comorbid conditions. However, patients with MS without these conditions also can experience significant fatigue, which is often described as a sensation of mental exhaustion, frequently occurring in the midafternoon. Lifestyle adjustments, such as improving sleep hygiene, getting regular exercise, and treating depression, can sometimes resolve this symptom. For those with refractory fatigue, stimulant medications can be used. The most common medications of this type used (off-label) in MS are modafinil, armodafinil, and amantadine. For fatigue that is refractory to these medications, amphetamine stimulants, such as methylphenidate, also can be considered.

A 51-year-old woman is evaluated for a 1-year history of daily afternoon fatigue that necessitates frequent naps and impairs her concentration at the office, where she works as a lawyer. Lifestyle adjustments, such as improving sleep hygiene, getting regular exercise, yoga, and vitamin supplementation have not resolved this symptom. She has a 5-year history of multiple sclerosis. Medications are glatiramer acetate and a vitamin D supplement. On physical examination, vital signs are normal. Depression screening is negative. The remainder of the physical examination is noncontributory. Results of laboratory studies, including hemoglobin and serum thyroid-stimulating hormone levels, are unremarkable. Which of the following is the most appropriate management?

Ovarian teratoma. An autoimmune condition termed anti-N-methyl-D-aspartate receptor (anti-NMDAR) antibody encephalitis has emerged as an increasingly common cause of encephalitis. Anti-NMDAR antibody encephalitis is associated with ovarian teratomas in more than 50% of patients with the disease because of production of an antibody to a tumor protein that cross-reacts with neuronal tissue. The diagnosis is suggested by the presence of choreoathetosis, psychiatric symptoms, seizures, and autonomic instability and is confirmed by detection of anti-NMDAR antibody in serum. This patient has developed new-onset psychiatric disease and a movement disorder. These symptoms raise suspicion of paraneoplastic or autoimmune encephalitis, and testing confirms the presence of the anti-NMDAR antibody in the serum. Patients are evaluated with CT and/or transvaginal ultrasound. Treatment includes removal of the teratoma to eradicate the immune stimulus and immunosuppression with glucocorticoids or intravenous immune globulin.

A 51-year-old woman is evaluated in the emergency department for a 1-month history of new-onset abnormal movements, paranoia, hallucinations, and progressive confusion. She was healthy before onset of symptoms and takes no medication. On physical examination, vital signs are normal. The patient is alert but oriented to person and place only, recalls none of three objects after 5 minutes, requires constant redirection to follow commands and sustain attention, is having ongoing visual and auditory hallucinations, and exhibits intermittent slow, writhing (choreiform) movements of the arms. Muscle strength is normal, as are sensation and deep tendon reflexes. Results of standard laboratory studies, including a complete blood count and comprehensive metabolic panel, are unremarkable. Results of brain MRI and lumbar puncture are normal. Subsequent testing for serum anti-N-methyl-D-aspartate receptor antibody is positive. Which of the following is the most likely diagnosis?

Cognitive rehabilitation. Cognitive dysfunction occurs in at least 50% of patients with multiple sclerosis (MS). The most common deficits involve short-term memory, processing speed, and executive function. Cognitive disability has a significant effect on the employability of patients with MS and can reduce their overall quality of life. To this point, however, no pharmaceutical agent has been shown to improve these symptoms in patients with MS. In contrast, cognitive rehabilitation approaches, such as the development of accommodative strategies and training with challenging cognitive tasks, have shown this benefit and should be pursued in this patient.

A 55-year-old man is evaluated for increasing difficulty keeping track of tasks and performing his job adequately. The patient works as an accountant and lately has been making frequent calculation errors. He also has noticed some word-finding difficulties and marked difficulty with short-term memory. Symptoms have progressed over the past 18 months but recently have become more prominent. The patient has a 15-year history of multiple sclerosis and also has depression, which has been in remission. Medications are glatiramer acetate, vitamin D3, and fluoxetine. On physical examination, vital signs are normal. On neurologic examination, the patient can recall only one of three objects at 3 minutes, skips "August" when reciting the months backward, and makes one error when subtracting serial sevens. Which of the following is the most appropriate treatment?

POEMS syndrome. The most likely diagnosis is POEMS syndrome (polyneuropathy, organomegaly, endocrinopathy, monoclonal gammopathy, and skin changes). POEMS syndrome is characterized by the presence of a monoclonal plasma cell disorder, peripheral neuropathy, and one or more of the following: osteosclerotic myeloma, Castleman disease (angiofollicular lymph node hyperplasia), elevated serum vascular endothelial growth factor, organomegaly, endocrinopathy, edema, typical skin changes, and papilledema. His clinical presentation, including splenomegaly, skin lesions (hyperpigmentation and angiomas), endocrine disease (hypothyroidism), peripheral neuropathy, and λ monoclonal gammopathy support the diagnosis of POEMS syndrome. POEMS syndrome is typically secondary to an underlying cancer; identification and treatment of the underlying cancer leads to improvement of the neuropathy.

A 52-year-old man is evaluated for a 1-year history of progressive weakness that began as right foot drop and bilateral tingling in the feet. Within the past 2 months, the patient has developed progressive weakness, which makes walking difficult; he also notes weakness in the hands and burning below the knees but no autonomic symptoms. He has hypothyroidism treated with levothyroxine. On physical examination, vital signs are normal. Motor strength is 4/5 in the intrinsic hand and quadriceps muscles and 3/5 in the tibialis anterior and gastrocnemius muscles; bulbar and facial muscle strength is normal. Deep tendon reflexes are absent in the lower extremities. Sensory perception of vibration is severely impaired at the knees. Pinprick testing shows reduced sensation below the ankles. Splenomegaly is present, as are patchy areas of hyperpigmentation and scattered angiomas on the trunk. Gait is broad based and wobbly, and a Romberg test has positive results. Serum immunofixation reveals a λ light chain monoclonal protein. Needle electromyography reveals a demyelinating sensorimotor polyneuropathy. Which of the following is the most likely diagnosis?

Epidural blood patch (EBP). Spontaneous intracranial hypotension classically presents with an orthostatic headache. The interval between postural change and headache development is highly variable. When the headache duration is weeks to months, the orthostatic component may fade completely. Presentation may be thunderclap (maximum onset within 1 minute) or subacute. Intracranial hypotension is the result of cerebrospinal fluid (CSF) leakage that can arise as a result of lumbar puncture, surgery, or trauma or can occur spontaneously. Associated features include tinnitus, diplopia, neck pain, nausea, photophobia, and phonophobia. Clinical examination findings are typically normal but occasionally reveal a "falsely localizing" abducens nerve (cranial nerve VI) palsy. Female sex, middle age, and connective tissue disorders are risk factors. Brain MRI with contrast is abnormal in 80% of affected patients, with possible findings of diffuse nonnodular pachymeningeal enhancement, cerebellar tonsillar abnormalities, and subdural fluid collections.

A 52-year-old woman is evaluated for a 3-week history of new-onset daily headaches. The pain is absent nocturnally and on awakening but starts within 15 minutes of the patient's arising from bed and becomes progressively severe throughout the day. The headache is global, steady, and (when severe) associated with photophobia and mild nausea. Intermittent bilateral tinnitus and brief episodes of horizontal diplopia also have occurred. The pain improves within 15 to 20 minutes of the patient's lying down. Analgesic agents have been unhelpful. She has no other medical problems. On physical examination, vital signs are normal; BMI is 26. Partial right abducens nerve (cranial nerve VI) palsy is noted. An MRI of the brain shows diffuse nonnodular pachymeningeal enhancement, a cerebellar tonsillar descent of 3 mm, and clinically insignificant bilateral subdural fluid collections. Which of the following is the most appropriate first step in management?

Citalopram. This patient should be given the selective serotonin reuptake inhibitor citalopram to control her obsessive-compulsive behaviors. Early changes in social behavior and personality are the defining characteristics of behavioral-variant frontotemporal dementia (FTD). Apathy, diminished interest, loss of empathy, lack of initiative, increased emotionality, disinhibition, euphoria, impulsivity, changes in eating behaviors, hyperorality, and compulsiveness are the most common symptoms reported by families. Other changes include irritability, aggression, verbal abuse, hypomania, and restlessness. The treatment of behavioral-variant FTD is symptom based and should target the most troubling manifestations of the disorder. This patient's obsessive-compulsive tendencies not only have had embarrassing consequences but have resulted in a confrontation with family members. Selective serotonin reuptake inhibitors, such as citalopram, have the potential to alleviate these symptoms. Tricyclic antidepressants may also have this effect.

A 53-year-old woman is brought to the office by her husband for follow-up evaluation of behavioral-variant frontotemporal dementia, which was diagnosed 9 months ago. Her clinical symptoms of occasional aphasia, minor memory impairment, behavioral disinhibition, and obsessive-compulsive behaviors have worsened since diagnosis and have become disruptive. She is currently estranged from her son and his family after bringing a large bag filled with plastic bottles collected from his neighbors' trash cans to her granddaughter's birthday party 2 weeks ago. Her husband asks about possible medications to control her symptoms.Which of the following is the most appropriate medication to recommend?

Intravenous heparin. Venous thromboembolism (VTE) is a common complication in patients with brain tumors, occurring in up to 30% of patients with high-grade glioma. Risk of VTE is correlated with higher-grade malignancies and is associated with release of the potent procoagulant tissue factor. Other factors, including immobilization and recent surgery, increase the risk. The risk of intracranial hemorrhage with the use of anticoagulants complicates the management of VTE in patients with brain tumor, including patients undergoing brain surgery. Although brain tumors have a risk of hemorrhage, therapeutic anticoagulation is generally considered safe. Intravenous heparin is the best choice for this patient because it has a short half-life and is reversible should hemorrhage occur. Although evidence does not support routine use of preanticoagulation neuroimaging to assess for hemorrhage, noncontrast head CT can be considered and is the most cost-effective test in this situation.

A 55-year-old woman is evaluated in the hospital after diagnosis of pulmonary embolism. She was admitted to the hospital 3 days ago for evaluation of a brain mass with imaging features characteristic of glioblastoma. Twenty-four hours after brain biopsy, the patient developed chest pain and dyspnea. Imaging confirmed a filling defect in the right pulmonary artery. On physical examination, blood pressure is 110/60 mm Hg, pulse rate is 100/min, respiration rate is 18/min, and oxygen saturation is 93% with the patient receiving 2 L/min of nasal oxygen. Cardiopulmonary examination shows mild tachycardia but is otherwise normal. Right upper and lower facial numbness and right arm weakness and numbness are noted. Biceps, triceps, and brachioradialis reflexes are increased on the right, as is right-sided hyperreflexia. An extensor plantar response is present on the right. Results of laboratory studies show a serum creatinine level of 1.7 mg/dL (150 µmol/L). Which of the following is the most appropriate treatment?

Lisinopril. The patient had an intracerebral hemorrhage in the thalamus due to hypertension. Given his age and hypertension, the hemorrhage is unlikely to have had a secondary cause. During acute hospitalization for an intracerebral hemorrhage, lowering the blood pressure acutely is recommended, although achieving systolic blood pressure levels of less than 140 mm Hg is associated with increased adverse events. After the initial hospitalization, however, the risk of recurrent ischemic and hemorrhagic stroke remains increased if the blood pressure is higher than 130/80 mm Hg, according to secondary prevention trials of stroke examining the effectiveness of ACE inhibitors. There is no contraindication to starting an ACE inhibitor, such as lisinopril, which reduces the risk of ischemic and hemorrhagic stroke regardless of blood pressure levels.

A 56-year-old man comes to the office for a follow-up evaluation after hospitalization for an intracerebral hemorrhage in the right thalamus. He reports feeling generally fatigued but has had no symptoms, such as depressed mood, since discharge. The patient has hypertension. His only medication is chlorthalidone. On physical examination, blood pressure is 136/78 mm Hg, pulse rate is 68/min, and respiration rate is 12/min; BMI is 32. Neurologic examination reveals a left-sided loss of sensation and 4/5 muscle strength with mildly increased tone throughout the left side. Results of laboratory studies show a serum creatinine level of 1.1 mg/dL (97.2 µmol/L), an LDL cholesterol of 86 mg/dL (2.23 mmol/L), and a hemoglobin A1c value of 5.9%. His calculated 10-year atherosclerotic heart disease risk for a major cardiovascular event is 4.3%. Initiation of which of the following medications is the most appropriate next step?

Administration of low molecular weight heparin. He has severe hemiparesis. Two poststroke CT scans obtained 48 hours apart show no evidence of active bleeding. The hemiparesis places him at high risk of deep venous thrombosis (DVT), which is the leading complication of hospitalization for stroke. DVT is found in 11% of immobile patients at 1 week after stroke and pulmonary embolism in about 1% of patients with stroke. Prophylaxis with low-molecular-weight heparin can be started 48 hours after hemorrhagic stroke in patients with no evidence of active bleeding on imaging. Before that time, DVT prophylaxis is possible with external sequential compression devices.

A 56-year-old man is hospitalized 8 hours after developing difficulty speaking and left-sided weakness. An emergent CT scan of the head shows an acute right thalamic hemorrhage with no hydrocephalus or intraventricular hemorrhage; a follow-up CT scan obtained 48 hours later shows no changes. The patient also has hypertension and type 2 diabetes mellitus. Medications before admission were lisinopril, aspirin, and glipizide. On hospital admission, hydrochlorothiazide and lisinopril are continued, aspirin and glipizide are discontinued, and insulin is initiated. On physical examination, blood pressure is 186/94 mm Hg, pulse rate is 74/min and regular, and respiration rate is 12/min. Left facial weakness with dysarthria and left-sided body weakness with no discernible movement are noted. An electrocardiogram shows sinus rhythm without acute changes. Which of the following is the most appropriate next step in management?

Multiple system atrophy. The combination of parkinsonism, cerebellar ataxia, dysautonomia, and early postural instability and falls in this patient is most consistent with multiple system atrophy, a Parkinson-plus syndrome. Typical autonomic deficits include orthostatic hypotension and urinary symptoms. Anosmia and acting out of dreams during sleep are suggestive of multiple system atrophy and Parkinson disease but not of progressive supranuclear palsy. A trial with levodopa may provide limited or partial benefit for this patient.

A 56-year-old woman is evaluated for a 1-year history of tremor that is most notable on the right. She also has developed increasing problems with balance resulting in several falls, especially when she stands from a seated position or turns abruptly. The patient exhibits occasional shouting and arm flailing during sleep. On physical examination, blood pressure is 128/75 mm Hg sitting and 105/70 mm Hg standing, pulse rate is 65/min sitting and 75/min standing, and respiration rate is 22/min. Decreased facial expression, full range of extraocular movements with dysmetric saccades, and hypophonic speech are noted. A low-amplitude resting tremor is more prominent on the right; the patient also has a more symmetric bilateral postural and kinetic tremor. On finger-to-nose testing, movements are slow and usually miss the target. Repetitive finger tapping movements become progressively smaller in amplitude. Gait is wide based, with frequent veering to the side, and the patient comes close to falling whenever she turns; gait speed is normal, but arm swing is decreased. The pull test results in the patient falling backward, and she is unable to walk a straight line. An MRI of the brain is unremarkable. Which of the following is the most likely diagnosis?

No further testing or treatment is necessary. This patient most likely has periodic limb movements of sleep, a disorder characterized by periodic leg kicks, often with a stereotyped triple-flexion phenomenology that repeats periodically during sleep. This condition can occur in otherwise healthy persons or be associated with sleep disorders, such as restless legs syndrome, sleep apnea, narcolepsy, and others. If another sleep disorder is present, treatment targeting the associated disorder also improves the periodic limb movements. If no associated sleep disorder is present (as is suggested in this patient by history and polysomnographic findings), treatment is only necessary if limb movements cause sleep fragmentation (brief arousals that occur during a sleep period). In the absence of such issues, no further testing or treatment is recommended.

A 56-year-old woman is evaluated for a 6-month history of nocturnal leg movements. According to her husband, she frequently kicks her legs during sleep but does not exhibit vocalizations or complex movements while asleep. The patient is unaware of these movements and has not had any sensory discomfort or urge to move the legs. She snores at night but has had no sudden loss of consciousness, sleep attacks, or excessive daytime sleepiness. Results of physical examination and laboratory studies are unremarkable. A polysomnogram shows good sleep efficiency, a low apnea-hypopnea index, and absence of motor activity during the rapid eye movement phase of sleep; no sleep fragmentation is evident, despite frequent leg movements. A video recording of her movements reveals slow flexion movements of the ankles, knees, and hips that repeat every 20 seconds in a stereotyped manner. Which of the following is the most appropriate next step in management?

Urodynamic Testing. Her symptoms and history of multiple sclerosis (MS) suggest the presence of a neurogenic bladder. Given the mix of symptoms of urgency and frequency (symptoms of an overactive, spastic bladder) plus hesitancy and retention (symptoms of a hypotonic bladder or overactive sphincter), the patient has aspects of both hypertonic and hypotonic bladder. She also may have bladder-sphincter dyssynergia, in which the contraction of the bladder wall is not properly timed with relaxation of the urinary sphincter. Management of complicated forms of neurogenic bladder involves proper diagnosis through urodynamic testing (which will guide medication choices), potential use of urinary catheterization, and monitoring of postvoid residuals while the patient is treated to avoid urinary retention. Involvement of a urologist is often required.

A 59-year-old woman comes to the office for a follow-up evaluation of a 2-year history of worsening urinary urgency and frequency. Episodes of urinary incontinence have occurred more often in the past 6 months because she cannot walk fast enough to get to the bathroom in time. Recently, she also has experienced intermittent urinary hesitancy and a frequent feeling of incomplete emptying, sometimes requiring changing body position or manual pelvic pressure. The patient has a 5-year history of secondary progressive multiple sclerosis. She takes no disease-modifying therapy. Her only medication is a vitamin D supplement. Physical examination findings are unremarkable. Urinalysis results are normal. Which of the following is the most appropriate next step in management?

Nimodipine. She has an aneurysmal subarachnoid hemorrhage (SAH) and is at high risk for neurologic decline from cerebral vasospasm. The risk of vasospasm is greatest 5 to 10 days after SAH onset. Nimodipine is indicated for all nonhypotensive patients with SAH and is associated with improved neurologic outcomes and survival. Because nimodipine may not directly reduce the vasospasm, additional monitoring with repeat neurologic examinations and transcranial Doppler ultrasonography is indicated in a specialized ICU. The presence of vasospasm is suggested by worsening findings on neurologic examination and can be confirmed with CT angiography or catheter angiography, with the latter test having the additional benefit of providing endovascular therapy.

A 61-year-old woman is admitted to the ICU 1 hour after having a subarachnoid hemorrhage (SAH). An emergent CT scan of the head showed an SAH, and a cerebral angiogram revealed a 6-mm rupture of the anterior communicating artery. The patient has hypertension treated with amlodipine. On physical examination, vital signs are stable, and oxygen saturation is 96% with the patient breathing ambient air. She is somnolent. Nuchal rigidity is present. Subhyaloid hemorrhages are seen on funduscopic examination. No motor or sensory deficits are present. Administration of which of the following medications is the most appropriate treatment?

Cerebrospinal fluid (CSF) analysis. is likely to provide the most specific information about whether Alzheimer disease is the cause of this patient's mild cognitive impairment (MCI). This test can measure levels of soluble Aβ42 peptide and soluble tau protein, which can help identify a specific pattern consistent with a diagnosis of Alzheimer disease, namely, decreased Aβ42 and increased tau and p-tau levels. In a patient with MCI, these levels have a greater than 80% sensitivity and specificity in identifying Alzheimer disease as the cause of cognitive symptoms. With the emphasis of clinical trials targeting the earliest stages of Alzheimer disease, the use of CSF biomarkers can assist in the diagnosis and prognosis of the underlying cause of MCI and in identifying patients who could qualify for clinical trials.

A 64-year-old man comes to the office for a follow-up evaluation of mild cognitive impairment (MCI), which was diagnosed 6 months earlier after he scored 25/30 (normal, ≥26) on the Montreal Cognitive Assessment. The patient reports no worsening of symptoms. He now scores 24/30 on a repeat Montreal Cognitive Assessment, scoring 0/5 on the recall section. The patient wants to know if there are additional tests that would indicate whether his symptoms are caused by Alzheimer disease. A brain MRI obtained at the time of diagnosis was reported as normal. Which of the following diagnostic studies is most likely to establish a diagnosis?

Huntington disease. His history of psychiatric disease, impulsivity, and substance abuse and clinical examination findings of executive dysfunction, dysarthria, incoordination, and ataxia are consistent with Huntington disease. His nonsuppressible flowing and variable movements are consistent with chorea, a major manifestation of Huntington disease. A positive family history is common in patients affected by this autosomal dominant disorder, but in its absence, a family history of associated conditions is suggestive. His father's prominent psychiatric disease may have led to suicide before onset of the motor features of Huntington disease; young-onset parkinsonism is a common presentation in younger patients with the disease. Tetrabenazine and deutetrabenazine are the most appropriate treatments of chorea in this disorder.

A 64-year-old man is evaluated for an abnormal gait and twitching. He is brought to the office by his wife, who reports that the patient has depression and that he has abused drugs and alcohol for the past 15 years; she further states that within the past year, he has developed impulsivity, imbalance, and incoordination and often appears restless. His father had major depressive disorder and committed suicide at age 60 years. On physical examination, vital signs are normal. Orientation, language, memory, and attention are intact. Speech is dysarthric. The patient cannot perform multistep tasks and cannot sit still on command. He exhibits frequent and variable flowing movements of the limbs and trunk that he incorporates into purposeful gestures. Reflexes are brisk, and gait is rapid, wide based, and uncoordinated. Which of the following is the most likely diagnosis?

CT angiography of the head. She has an acute ischemic stroke and was appropriately treated within 3 hours of symptom onset with intravenous recombinant tissue plasminogen activator (alteplase). The neurologic examination was consistent with an acute occlusion of the left intracranial internal carotid artery or middle cerebral artery. The patient's atrial fibrillation and subtherapeutic INR make a cardioembolic stroke subtype likely. Patients with an ischemic stroke and large-vessel occlusion have low recanalization rates with intravenous thrombolysis, and recently completed clinical trials have shown a clinical benefit from the addition of endovascular therapy, such as embolectomy, among carefully selected patients. The first step in patient selection requires the presence of a large vessel occlusion on vessel imaging, which is most quickly seen with CT angiography.

A 64-year-old woman is evaluated in the emergency department 45 minutes after sudden onset of right-sided weakness and the loss of the ability to speak. An emergent noncontrast CT of the head shows no hemorrhage or early signs of infarct. The patient also has hypertension and atrial fibrillation. Medications are hydrochlorothiazide and warfarin. On physical examination, blood pressure is 158/78 mm Hg, and pulse rate is 72/min and irregularly irregular. Global aphasia, left-gaze preference, right hemiparesis, and loss of pain sensation on the right side are noted. Results of laboratory studies show an INR of 1.3. The patient receives intravenous recombinant tissue plasminogen activator (alteplase) 1 hour after symptom onset. Blood pressure is now 168/86 mm Hg, but other vital signs are unchanged, as are results of repeat neurologic examination. Which of the following is the most appropriate next step in management?

Continuous (24-hour) electroencephalography (EEG). He has fluctuating mental status of unknown origin in the setting of a critical illness, sepsis, and multiorgan dysfunction. These findings raise suspicion of nonconvulsive status epilepticus (NCSE), which is intermittent electrical seizure activity without clinically evident seizure activity. A 20-minute EEG usually is inadequately sensitive to capture seizures, particularly if they are occurring intermittently. NCSE also is increasingly diagnosed in critically ill comatose or stuporous patients with acute neurologic or medical conditions who have not had a convulsive seizure. Similar to NCSE following CSE, NCSE in critically ill populations also is associated with increased morbidity and mortality and requires prompt attention and intervention. The diagnostic test of choice is continuous (24-hour) EEG monitoring. Most critically ill patients with nonconvulsive seizures have their first detectable seizure with EEG monitoring within 24 hours of initiating the recording, but an additional 12% have detectable seizures with 48 hours of continuous monitoring. Cefepime has been known to cause encephalopathy, coma, and status epilepticus in patients with or without epilepsy, especially those with acute kidney injury, and its use should raise the clinician's suspicion of NCSE.

A 65-year-old man is evaluated in the ICU for a 24-hour history of altered mental status with a fluctuating level of consciousness. He was admitted to the hospital 5 days ago for urosepsis and acute kidney injury and developed acute respiratory distress syndrome on hospital day 3. The patient is currently intubated, mechanically ventilated, and receiving continuous hemodialysis. Medications are cefepime, norepinephrine, and fentanyl. On physical examination, temperature is 38.4 °C (101.1 °F), blood pressure is 105/71 mm Hg, pulse rate is 108/min, and respiration rate is 12/min; FIO2 is 0.9. The patient opens his eyes to voice but does not fixate on the examiner or follow commands. Pupils are reactive, and gag and corneal reflexes are present. All limbs move intermittently but not on command. The patient withdraws from painful stimuli in all four limbs. Intermittent twitching of the shoulders and eyelids is noted. Results of laboratory studies show a serum creatinine level of 5.4 mg/dL (477 µmol/L). Glucose, calcium, and electrolyte levels are within normal limits. A 20-minute electroencephalogram shows generalized slow activity, a nonspecific finding compatible with encephalopathy, but no evidence of seizure activity. A head CT scan is normal. Which of the following is the most appropriate next step in management?

Physical therapy. Spinal cord compression can result from acute or chronic causes. Evaluation and confirmation of suspected acute spinal cord compression with appropriate neuroimaging studies should occur in an urgent manner. Immediate treatment may be necessary to prevent severe and irreversible neurologic injury. Patients with chronic spinal stenosis due to osteoarthritic degenerative spinal disease frequently have chronic myelopathic symptoms, most often involving the cervical and lumbar spines. This patient has chronic symptoms of cervical stenosis due to multilevel disc disease. Most patients with chronic cervical and lumbar stenosis respond well to conservative measures, such as physical therapy and pain control. However, those with symptoms of more moderate to severe disease who also have signs of myelopathy on examination, such as progressive leg weakness, spasticity, distal numbness, and bladder impairment, may require surgical intervention. The lack of these findings in this patient supports the use of conservative measures, such as physical therapy.

A 68-year-old woman is evaluated for a 1-year history of neck stiffness and dull, achy neck pain. She also notes intermittent difficulty with dexterity while performing fine motor tasks at the hair salon where she works. Medications are ibuprofen as needed. On physical examination, vital signs are normal. Range of motion of the neck is limited because of pain and stiffness. Fine finger movements exhibit subtle slowness. Reflex examination findings are normal, including a plantar flexor response. Muscle strength is 5/5 throughout. Gait is normal. An MRI of the cervical spine shows multilevel cervical stenosis that is worst at C4/5 and C6/7. There is moderate deformation of the cord, but no signal change in the cord is noted. Which of the following is the most appropriate next step in management?

Amyotrophic lateral sclerosis. Diagnosis requires fulfillment of positive diagnostic criteria and exclusion of mimics. Positive diagnostic criteria for ALS include the presence of clinical upper (hyperreflexia, Hoffman sign, clonus) and lower (atrophy, fasciculation, weakness) motor neuron signs and electromyographic lower motor neuron signs in two (probable ALS) or more (definitive ALS) body regions (upper and lower limbs, paraspinal, bulbar). Because hyperthyroidism can lead to a combination of upper and lower motor neuron findings that may mimic ALS, it must be excluded before diagnosis. Other important mimics include structural brain and cervical spinal lesions, vitamin B12 and copper deficiency, Lyme disease, and hyperparathyroidism.

A 70-year-old man is evaluated for a 6-month history of right upper extremity weakness, intermittent painful spasms in both calves, and occasional brief muscle twitches. The patient reports right arm and hand weakness and says that for the past 2 weeks, his right foot drags when he walks quickly. He has had no numbness, sensory loss, paresthesia, or problems with swallowing, sphincter control, or coordination. On physical examination, vital signs are normal. Weakness and muscle wasting is noted in the thumb abductor and intrinsic hand muscles on the right side. Spontaneous muscle twitches are present in the bilateral thumb adductor, gastrocnemius, and paraspinal muscles. Minor weakness of right ankle dorsiflexion, a right Hoffman sign, and right foot clonus is noted. Deep tendon reflexes are brisk throughout. Cranial nerves are intact, as is sensory perception. Electromyography and nerve condition studies reveal evidence of lower motor neuron involvement in the upper and lower extremities and paraspinal muscles. MRIs of the brain and cervical spine are unremarkable. Which of the following is the most likely diagnosis?

Lamotrigine. He has a history of focal seizures with altered awareness (formerly known as complex partial seizures) and also has had a generalized tonic-clonic seizure. Only a few antiepileptic drugs (AEDs) (namely, lamotrigine, levetiracetam, topiramate, valproic acid, and zonisamide) are considered broad-spectrum agents and can be used to treat both generalized and partial epilepsy syndromes. Other narrow-spectrum AEDs (such as carbamazepine, gabapentin, oxcarbazepine, phenobarbital, phenytoin, and pregabalin) have the potential to exacerbate seizures in patients with generalized epilepsy. Lamotrigine is the most appropriate treatment because of the good evidence of its safety and effectiveness in epilepsy with focal seizures, especially in older patients, and is effective in generalized seizures. Lamotrigine is unlikely to cause cognitive dysfunction and does not affect blood counts, the liver, the kidney, or electrolytes. Although lamotrigine is associated with Stevens-Johnson syndrome, the drug can be titrated slowly to minimize risk; this patient's seizures are infrequent enough to allow for a 6- to 8-week titration. Although all the other drugs listed are likely effective in controlling seizures, lamotrigine also has a lower risk of adverse effects.

A 71-year-old man is evaluated for a 2-year history of episodes of staring, confusion, and repetitive left-arm movements (grabbing at his clothes or face). These events occur every 2 to 3 months, last approximately 2 minutes, and typically are followed by a 15-minute period of disorientation and fatigue. On one occasion, the episode was followed immediately by the patient's falling down, experiencing shaking of the entire body, and having urinary incontinence. He also has hypertension, nephrolithiasis, and mild cognitive impairment. Medications are hydrochlorothiazide and metoprolol. All physical examination findings, including vital signs, are normal. A routine electroencephalogram and brain MRI are normal. Which of the following is the most appropriate treatment?

Myasthenia gravis. can present with pronounced weakness of cervical or bulbar muscles. Fluctuation in weakness, with a fatigable pattern that worsens later in the day, suggests this neuromuscular junction disorder. In the setting of probable myasthenia gravis, seronegative status for acetylcholine receptor antibodies should trigger testing for muscle-specific kinase (MuSK) antibodies, which are positive in half of seronegative patients. Weakness of cervical extension, such as this patient exhibits, is a hallmark of MuSK antibody-positive myasthenia gravis. Although nerve conduction studies using a special protocol for repetitive stimulation can reveal a pattern of decrementing weakness in neuromuscular junction disorders, this protocol is not part of routine electromyography. Therefore, if suspicion for myasthenia gravis is high, a repetitive stimulation protocol should be specifically requested.

A 71-year-old woman is evaluated for difficulty holding her head upright. She notes that her head feels heavy and reports intermittent difficulty with swallowing and speech that is worse in the evening. She has had no pain, sensory changes, weakness in the extremities, or cognitive or visual symptoms. She has no other medical problems and takes no medication. On physical examination, vital signs are normal. Speech is mildly dysarthric. Cervical extension is weak. No ptosis, ophthalmoplegia, sensory deficit, or weakness in the extremities is noted. Results of laboratory studies show a normal serum creatine kinase level; no acetylcholine receptor antibodies are detected. Findings from routine nerve conduction and needle electromyography studies of the limbs are unremarkable, but a repetitive stimulation protocol reveals a decremental response. An MRI of the brain is normal. Which of the following is the most likely diagnosis?

Brain MRI. His development of a new headache after age 50 years, use of an anticoagulant, and progressive headache pattern are all red flags indicating a potential secondary headache condition. Other red flags for secondary headache include first or worst headache, abrupt-onset or thunderclap attack, episode associated with neurologic symptoms lasting more than 1 hour, alterations in consciousness, abnormal physical examination findings, and onset after exertion, sex, or Valsalva maneuvers. Brain MRI is the most appropriate diagnostic study for patients with a suspected secondary headache and should be performed before additional testing is considered.

A 72-year-old man is evaluated for a 6-month history of headaches that have gradually increased in frequency and have been daily and constant for the past 6 weeks. The pain is bilateral, occipital, and described as a steady "pulling" sensation of mild to moderate intensity. Fatigue and neck discomfort are present. He has had no associated nausea, photophobia, phonophobia, or neurologic symptoms. The patient has atrial fibrillation treated with warfarin and verapamil and no other personal or family history of headache. On physical examination, blood pressure is 128/88 mm Hg and pulse rate is 88/min and irregular. Decreased cervical range of motion is noted. Neurologic examination findings are unremarkable. Results of laboratory studies show a normal erythrocyte sedimentation rate and complete blood count, a therapeutic INR, a normal comprehensive metabolic profile, and a normal serum level of C-reactive protein. Which of the following is the most appropriate next step in management?

Outpatient cardiac telemetry. She has a cryptogenic infarct with no clear source identified on arterial imaging, no evidence of atrial fibrillation (AF) or other high-risk embolic cause, and a stroke location that is not typical for lacunar infarcts. Accumulating data on patients with cryptogenic stroke indicate that an evaluation for AF with an outpatient rhythm monitor may yield a new diagnosis of AF in almost one third of patients. Given the high risk of recurrent stroke associated with AF, additional outpatient evaluation is warranted. The various options available for monitoring include mobile outpatient cardiac telemetry, 24-hour electrocardiographic monitoring, transtelephonic and event monitors, and implantable subcutaneous devices. Longer monitoring results in a higher diagnostic yield. In one study comparing 30-day monitoring to 24-hour monitoring, atrial fibrillation lasting for at least 30 seconds was found in 16.1% of patients monitored for 30 days vs. 3.2% of patients monitored for 24 hours, and AF lasting for at least 2.5 minutes was found in 9.9% of patients with prolonged monitoring vs. 2.5% of patients with 24-hour monitoring.

A 72-year-old woman is evaluated in the emergency department 5 hours after developing difficulty speaking and facial weakness on the right. She takes no medication. On physical examination, vital signs are normal. The patient is awake and attentive. Spontaneous speech is slow. Right-sided facial weakness and dysarthria are noted. Hemoglobin level, platelet count, and coagulation profile are within normal limits. An electrocardiogram is normal. A CT scan of the head shows an acute left frontal ischemic stroke. A carotid duplex ultrasound reveals less than 40% stenosis in both internal carotid arteries; a transcranial Doppler ultrasound is normal. A transthoracic echocardiogram shows an ejection fraction of 50% but is otherwise unremarkable. Aspirin and rosuvastatin are initiated, and the patient is admitted to the telemetry unit for 3 days, during which time she remains in sinus rhythm. Which of the following is the most appropriate next step in management?

Pull test. This patient's risk of falling backward is best determined by the pull test, which is the most sensitive predictor of risk of backward falls in Parkinson disease. During this test of postural stability, the examiner throws the patient off base by pulling backward on the shoulders; the test is considered positive if the patient topples into the examiner's arms or takes more than two corrective steps. Backward falls are often related to loss of postural reflexes and resultant postural instability. Additional factors, including insufficient control of motor symptoms, dyskinesia, and orthostatic hypotension, also can contribute to falls in Parkinson disease. In this patient, history and gait assessment did not reveal any interference with balance caused by freezing of gait or lower body dyskinesia, and he had no symptoms or findings suggestive of orthostatic hypotension.

A 76-year-old man is evaluated for recent worsening of balance. He has no dizziness or lightheadedness. The patient has an 8-year history of Parkinson disease treated with carbidopa-levodopa and entacapone. On physical examination, vital signs are normal; no orthostatic decrease in blood pressure is noted. The patient has masked facies, a resting tremor, and bradykinesia. Gait assessment reveals mild shuffling without freezing. Findings from cognitive, cerebellar, and sensory examinations are unremarkable. Which of the following is the most appropriate test to determine the patient's risk of falling backward?

Begin Treatment For Depression. This patient should undergo treatment for depression. Depression is common in older adults, although it is both underrecognized and undertreated. In these patients, depression can frequently masquerade as cognitive impairment or multiple somatic complaints, which often results in delayed diagnosis. The patient expresses concern about subjective cognitive changes. However, given his age, the fact that he recently assumed new responsibilities after losing his wife, the limited evidence of memory loss, and the normal findings on cognitive and neurologic examination, a significant cognitive disorder is not likely. His recent decreased appetite, problems falling asleep, and withdrawal from social activities suggest onset of a depressive disorder that is most likely associated with the loss of his wife. Depression is further suggested by the positive Geriatric Depression Scale (GDS) score of 9. The GDS has been validated in the older population. The GDS relies less on somatic symptoms and has a simplified yes-or-no format, which may be more conducive to screening those with possible cognitive impairment. Treatment of his depression with close follow-up evaluations is thus most advisable.

A 78-year-old man is evaluated for recent difficulty with memory. The patient has hypertension treated with hydrochlorothiazide. He reports decreased appetite, problems falling asleep, and withdrawal from social activities since his wife's death 13 months ago, at which time he had to assume several new responsibilities, including managing the finances and cooking. He is concerned because he received a missed payment notice on last month's utility bill and is worried that he is developing dementia; his 85-year-old brother has Alzheimer disease that was diagnosed 2 years ago. On a screening evaluation 6 months ago, the patient scored 28/30 (normal, ≥24) on a Mini-Mental State Examination, and his Geriatric Depression Scale Short Form score was 9 (normal, <6); he has lost 2.3 kg (5.1 lb) since that time. Physical examination findings, including vital signs and results of neurologic examination, are unremarkable. Results of laboratory studies, including a complete blood count and measurement of thyroid-stimulating hormone and vitamin B12 levels, are normal. Which of the following is the most appropriate next step in management?

Vitreous fluid sampling. He most likely has primary central nervous system lymphoma (PCNSL). PCNSL is a non-Hodgkin lymphoma that can affect any part of the central nervous system but commonly presents as a focal supratentorial lesion. Although PCNSL is commonly seen in patients with HIV infection, PCNSL is increasing in incidence among older, immunocompetent patients. An association with Epstein-Barr virus has been noted. Pathologic analysis, usually of a brain biopsy specimen, is required to make a diagnosis of PCNSL. Diffuse large B-cell lymphoma is typical. Cerebrospinal fluid (CSF) cytology can be diagnostic in 10% of patients, although repeated samples often are necessary. Ocular involvement in the vitreous or retina may be seen in 10% to 20% of patients and can be detected with a slit-lamp examination and confirmed by vitreous fluid sampling. In this patient, vitreous fluid collection with cytologic analysis may obviate the need for more invasive and potentially risky testing, such as brain biopsy. Despite being chemo- and radiosensitive, PCNSL typically recurs and has a generally poor prognosis.

A 79-year-old man is evaluated for a 2-month history of progressively worsening headaches, nausea, visual disturbance, and difficulty speaking. He also has hypertension and gastroesophageal reflux disease. Medications are lisinopril and omeprazole. On physical examination, vital signs are normal. Right oculomotor nerve (cranial nerve III) and bilateral abducens nerve (cranial nerve VI) palsies are noted, as is right upper and lower facial weakness. An MRI of the brain shows a well-demarcated, homogeneously enhancing, hyperintense lesion that is suspicious for primary central nervous system lymphoma in the brainstem extending from the left pons to the medulla. Mass effect but no edema is present. The lesion obstructs the cerebral aqueduct, and hydrocephalus with enlargement of the lateral and third ventricles is noted. Which of the following is the most appropriate next step in management?

Clonidine. The suppressible stereotyped neck movements and premonitory sensory cues are consistent with motor tics, and the history of other motor (eye rolling) and vocal (throat clearing) tics, childhood onset, duration of greater than 1 year, and comorbidity of obsessive-compulsive disorder (OCD) are consistent with the diagnosis of Tourette syndrome. Clonidine is a first-line agent used to treat Tourette syndrome. Anti-tic medications should be considered when tics interfere with education, daily function, or work. Other first-line medications include guanfacine, topiramate, and the dopamine-depleter agent tetrabenazine. Second-line treatments include antipsychotic agents (such as haloperidol), but their benefit should be weighed against risk of tardive dyskinesia.

An 18-year-old woman is evaluated for a 2-year history of repetitive fast movements of the neck that tilt the head to the right side. She reports that these movements are preceded by a feeling of discomfort in the right shoulder and that if she concentrates and taps the right foot immediately after this sensory cue, she usually can avoid the neck movements. She is a senior in high school and says she has not been able to use this method successfully in the classroom, especially during examinations, when she often experiences a cluster of movements that exhaust and distract her. Her mother notes that before onset of the neck symptoms, the patient used to roll her eyes and clear her throat frequently. She has obsessive-compulsive disorder treated with sertraline. On physical examination, vital signs are normal. During the examination, the patient displays rapid tilting movements of the head followed by rolling of the head and shoulder; these movements repeat several times in a stereotyped manner. The patient can suppress these movements on request. Which of the following is the most appropriate treatment?

Memantine. She has probable Alzheimer disease, moderate stage, given the degree of functional impairment she exhibits and her Mini-Mental State Examination score. The treatment of Alzheimer dementia is multifactorial and is symptom targeted. Both cholinesterase inhibitors (such as donepezil) and the N-methyl-D-aspartate receptor antagonist memantine are approved for treatment of moderate stages of Alzheimer disease. The cholinesterase inhibitors exert their effect by inhibiting the enzymes responsible for breaking down acetylcholine, thereby increasing the levels of acetylcholine in the neuronal synapse. No clinically significant difference in effectiveness has been shown between the cholinesterase inhibitors. Relative contraindications for their use include (but are not limited to) sick sinus syndrome, left bundle branch block, uncontrolled asthma, angle-closure glaucoma, and ulcer disease. Memantine is believed to reduce glutamate-mediated neurotoxicity in the central nervous system, is not associated with adverse cardiovascular effects, and thus is the most appropriate treatment for this patient.

An 84-year-old woman is evaluated for dementia. She lives alone and is brought to the office by her son, who reports first noticing gradually progressive symptoms 4 years ago when his mother began having episodes of memory loss and a tendency to repeat herself excessively. He also notes increasing hygiene-related problems and says his mother recently called him several times in the middle of the night and asked him to pick her up "so she can go home." She can no longer manage her finances, keep appointments, or remember to take her medications; she stopped driving 1 year ago after getting lost several times. She has sick sinus syndrome and hypothyroidism treated with levothyroxine. On physical examination, blood pressure is 155/60 mm Hg and pulse rate is 55/min and intermittently irregular. She scores 17/30 (normal, ≥24) on the Mini-Mental State Examination, but all other findings from neurologic examination are unremarkable. Results of laboratory studies and brain imaging are normal. Which of the following is the most appropriate treatment at this time?


Related study sets

Module 1 Lesson 2 Types of Network Topologies Main Ideas

View Set

EMTB CH 28 HW and Quiz Questions

View Set

Ch. 6 - Future and Present Values of Multiple Cash Flows

View Set

Ch. 01 Module 3: Sections 1.05-1.06

View Set

Basic Accounting for Partnership

View Set

Advanced Biology Photosynthesis/Respiration

View Set

The American Pageant Study Guide Chapter 8 APUSH

View Set

Chapter 4 - Cell structure and membranes

View Set